Administración     

Olimpiadas de Matemáticas
Página de preparación y problemas

OME Local
OME Nacional
OIM
OME Andalucía
Retos UJA
Selector
La base de datos contiene 1154 problemas y 775 soluciones.

XVIII Olimpiada Iberoamericana de Matemáticas — 2003

Sesión 1 —  Mar del Plata (Argentina), 16 de septiembre de 2003

Problema 338
  1. Se tienen dos sucesiones, cada una de 2003 enteros consecutivos, y un tablero de 2 filas y 2003 columnas. Decidir si siempre es posible distribuir los números de la primera sucesión en la primera fila y los de la segunda sucesión en la segunda fila, de tal manera que los resultados obtenidos al sumar los dos números de cada columna formen una nueva sucesión de 2003 números consecutivos.
  2. ¿Y si se reemplaza 2003 por 2004?
Sin pistas
solución 1info
Solución. La respuesta es afirmativa para 2003. Escribiendo los números de la primera fila como $a_k=a_0+k$ y los de la segunda como $b_k=b_0+k$ para $k\in\{1,\ldots,2003\}$, una forma de distribuir los números es la siguiente:
$a_{1002}$$a_{1001}$$a_{1000}$...$a_{1}$$a_{2003}$$a_{2002}$$a_{2001}$...$a_{1001}$
$b_{1}$$b_{3}$$b_{5}$...$b_{2003}$$b_{2}$$b_{4}$$b_{6}$...$b_{2002}$
Los números de la primera fila forman dos sucesiones decrecientes de números consecutivos, mientras que los de la segunda forman dos sucesiones crecientes de números que van saltando de dos en dos.

Veamos que la respuesta es negativa para 2004. Razonando por reducción al absurdo, supongamos que la respuesta es afirmativa. Escribiendo los números de la primera fila como $a_k=a_0+k$, los de la segunda fila como $b_k=b_0+k$ y las sumas como $c_k=c_0+k$ para $k\in\{1,\ldots,2004\}$, tenemos que las sumas de los números de estas filas están dadas por \begin{eqnarray*} \sum_{k=1}^{2004}a_k&=&2004a_0+\sum_{k=1}^{2004}k=2004a_0+\frac{2004\cdot 2005}{2}=2004\left(a_0+\frac{2005}{2}\right),\\ \sum_{k=1}^{2004}b_k&=&2004b_0+\sum_{k=1}^{2004}k=2004b_0+\frac{2004\cdot 2005}{2}=2004\left(b_0+\frac{2005}{2}\right),\\ \sum_{k=1}^{2004}c_k&=&2004c_0+\sum_{k=1}^{2004}k=2004c_0+\frac{2004\cdot 2005}{2}=2004\left(c_0+\frac{2005}{2}\right). \end{eqnarray*} Ahora bien, independientemente de la colocación de los $a_k$ y $b_k$, la suma de las dos primeras sumas ha de ser igual a la de la tercera, luego tenemos que \[a_0+b_0+\frac{2005}{2}=c_0.\] Esto es una contradicción ya que $a_0$, $b_0$ y $c_0$ son números enteros mientras que $\frac{2005}{2}$ no lo es.

Nota. En la demostración anterior puede suponerse sin perder generalidad que $a_0=b_0=0$, con lo que $a_k=b_k=k$ para todo $k$ y así simplificar ligeramente la notación.

Si crees que el enunciado contiene un error o imprecisión o bien crees que la información sobre la procedencia del problema es incorrecta, puedes notificarlo usando los siguientes botones:
Informar de error en enunciado Informar de procedencia del problema
Problema 767
Sean $C$ y $D$ dos puntos de la semicircunferencia de diámetro $AB$ tales que $B$ y $C$ están en semiplanos distintos respecto de la recta $AD$. Denotemos por $M$, $N$ y $P$ los puntos medios de $AC$, $DB$ y $CD$, respectivamente. Sean $O_A$ y $O_B$ los circuncentros de los triángulos $ACP$ y $BDP$. Demostrar que las rectas $O_AO_B$ y $MN$ son paralelas.
pistasolución 1info
Pista. Demuestra que $OO_AO_B$ y $OMN$ son semejantes, siendo $O$ el centro de la semicircunferencia. Te puede ser útil utilizar el teorema de Thales.
Solución. Las mediatriz de $AC$ pasa por $O_A$ y por el centro $O$ de la semicircunferencia y, de la misma forma, la mediatriz de $BD$ pasa por $O_B$ y por $O$. Por tanto, que $O_AO_B$ sea paralela a $MN$ equivale a que los triángulos $OO_AO_B$ y $OMN$ sean semejantes. Si trazamos perpendiculares a la recta $CD$ que pasan por $A$, $M$, $O_A$, $O_B$, $N$ y $B$ (en línea discontinua en la figura) y marcamos los puntos de intersección con $CD$ como $E$, $F$, $G$, $H$, $I$ y $J$, respectivamente, el teorema de Thales nos dice que \[\frac{OO_A}{GP}=\frac{OM}{PF},\qquad \frac{OO_B}{HP}=\frac{ON}{PI}.\] Utilizando que $G$ y $H$ son los puntos medios de $CP$ y $DP$ (ya que las perpendiculares por $O_A$ y $O_B$ son mediatrices de $CP$ y $DP$, respectivamente), las semejanzas anteriores se reescriben como \[\frac{OO_A}{OM}=\frac{CP}{2PF},\qquad \frac{OO_B}{ON}=\frac{DP}{2PI}.\] Como $CP=DP$ por ser $P$ el punto medio de $CD$, el problema estará resuelto si demostramso que $PF=PI$. Usando de nuevo que $CP=DP$, podemos reducir el problema aún más a probar que $PE=PJ$, pero esto último es sencillo ya que es consecuencia del teorema de Thales aplicado a las paralelas $BJ$, $OP$ y $AE$ que cortan a las rectas $AB$ y $AJ$, teniendo en cuenta que $O$ es el punto medio de $AB$.imagen
Si crees que el enunciado contiene un error o imprecisión o bien crees que la información sobre la procedencia del problema es incorrecta, puedes notificarlo usando los siguientes botones:
Informar de error en enunciado Informar de procedencia del problema
Problema 339
Pablo estaba copiando el siguiente problema:
Entre todas las sucesiones de 2004 números reales $\{x_0,x_1,x_2,\ldots,x_{2003}\}$ tales que \[x_0=1,\ 0\leq x_1\leq 2x_0,\ 0\leq x_2\leq 2x_1,\ \ldots,\ 0\leq x_{2003}\leq 2x_{2002},\] determine aquella para la que la siguiente expresión toma su mayor valor: \[S=\ldots\]
Cuando Pablo iba a copiar la expresión de $S$ le borraron la pizarra. Lo único que pudo recordar es que era de la forma $S=\pm x_1\pm x_2\pm\ldots\pm x_{2002}+x_{2003}$, donde el último término, $x_{2003}$, tenía coeficiente $+1$, y los anteriores tenían coeficiente $+1$ ó $–1$. Demostrar que Pablo, a pesar de no tener el enunciado completo, puede determinar con certeza la solución del problema.
pistasolución 1solución 2info
Pista. Supón que la sucesión que maximiza el valor de $S$ cumple que $x_k<2x_{k-1}$ para algún $k$ y llega a una contradicción encontrando otra sucesión para la que el valor de $S$ sea mayor.
Solución. Para simplificar la notación, escribamos $S=\sum_{n=1}^{2003}\epsilon_nx_n$, donde $\epsilon_n$ es el signo correspondiente a $x_n$, es decir $\epsilon_n=1$ ó $\epsilon_n=-1$. Supongamos además que estamos trabajando con la sucesión $\{x_n\}$ que maximiza $S$ y probemos que $x_n=2^n$ para todo $n$ independientemente del valor de los $\epsilon_n$ (observemos que esto es equivalente a demostrar que $x_n=2x_{n-1}$ para todo $n\in\{1,\ldots,2003\}$).
  • Veamos en primer lugar que no pueden haber términos nulos. Si $x_k=0$, entonces $x_n=0$ para todo $n\geq k$, luego podemos suponer que $x_k$ es el primer término nulo. Si consideramos la sucesión \[\overline{x}_n=\begin{cases}x_n,& \text{si }0\leq n\lt k,\\ 2\overline{x}_{n-1},&\text{si }k\leq n\leq 2003,\end{cases}\] (es decir, sustituimos todos los términos nulos por los máximos posibles) entonces \[S(\overline x)-S(x)=x_{k-1}\left(2^{2004-k}+\sum_{n=k}^{2002}\epsilon_n2^{n-k+1}\right)\geq x_{k-1}\left(2^{2004-k}-\sum_{n=k}^{2002}2^{n-k+1}\right)=2x_{k-1}>0,\] lo que nos dice que la sucesión $\overline x$ da un valor mayor para $S$, contradiciendo que habíamos supuesto que $x$ maximizaba el valor de $S$. En la desigualdad anterior, hemos usado que el valor mínimo se obtiene cuando todos los $\epsilon_n$ son negativos.
  • Supongamos ahora que todos los $x_n$ son positivos pero que existe $k$ tal que $x_k<2x_{k-1}$. Nuestro objetivo es llegar también a una contradicción.
    • Si $\epsilon_k=1$, entonces claramente la sucesión que resulta al sustituir $x_k$ por $2x_{k-1}$ mejora el valor de $S$, luego tenemos la contradicción buscada.
    • Si $\epsilon_k=-1$, entonces consideremos el primer $p>k$ tal que $\epsilon_p=1$ (que siempre existe ya que $\epsilon_{2003}=1$). Dado $\lambda=\frac{2x_{k-1}}{x_k}\gt 1$, podemos considerar la nueva sucesión \[\overline{x}_n=\begin{cases}2\overline{x}_{n-1},& \text{si }k\leq n\leq p,\\ x_n,&\text{para el resto de valores de }n.\end{cases}\] Entonces, podemos calcular \begin{eqnarray*} S(\overline x)-S(x)&=&2^{p-k+1}x_{k-1}-x_p-\sum_{n=k}^{p-1}(2^{n-k+1}x_{k-1}-x_n)=2x_{k-1}+\sum_{n=k}^{p-1}x_n-x_p\\ &\gt& 2x_k+\sum_{n=k+1}^{p-1}x_n-x_p\geq 2x_{k+1}+\sum_{n=k+2}^{p-1}x_n-x_p\geq\ldots\geq 2x_{p-1}-x_p\geq 0. \end{eqnarray*} En las últimas desigualdades hemos usado que $2x_{k-1}>x_k$ y $2x_{n-1}\geq x_n$ para el resto de valores de $n$. Esta desigualdad estricta nos dice que $\overline x$ mejora el valor de $S$ y volvemos a tener una contradicción.

Nota. En realidad, el propio problema nos da una pista fundamental para saber que la sucesión que maximiza ha de ser $x_n=2^n$ para todo $n$. Como la solución no depende de la elección de los signos $\epsilon_n$, también tiene que valer para todos positivos y ahí es claro que $S$ es máximo cuando todos los $x_n$ son lo más grandes posibles, esto es, $x_n=2x_{n-1}$ para todo $n$.

Solución. Este es un problema de programación lineal, ya que tenemos que maximizar una combinación lineal de los valores de $x_1,x_2,\ldots,x_{2003}$ con ciertas desigualdades lineales $0\leq x_k\leq 2x_{k-1}$. Los valores que maximizan $S$ serán uno de los vértices de la región de $\mathbb{R}^{2003}$ determinada por estas desigualdades (observemos que la región está acotada, luego la existencia de solución está garantizada). Usando que, en cuanto una variable es cero las sucesivas han de ser cero también, es fácil ver que dicha región tiene 2004 vértices, dados por \begin{eqnarray*} p_0&=&(0,0,0,0\ldots,0),\\ p_1&=&(2,0,0,0\ldots,0),\\ p_2&=&(2,4,0,0\ldots,0),\\ p_3&=&(2,4,8,0\ldots,0),\\ &\vdots&\\ p_{2003}&=&(2,4,8,16,\ldots,2^{2003}). \end{eqnarray*} Ahora será suficiente ver que $S(p_{2003})> S(p_k)$ para todo $k\in\{0,\ldots,2002\}$ independientemente del valor de los signos que definen a $S$. Si escribimos $S=x_{2003}+\sum_{n=1}^{2002}\epsilon_nx_n$, siendo $\epsilon_n\in\{-1,+1\}$ el coeficiente de $x_n$, obtenemos que, para todo $k\in\{0,\ldots,2002\}$, se cumple que \begin{eqnarray*} S(p_{2003})-S(p_k)&=&\left(2^{2003}+\sum_{n=1}^{2002}\epsilon_n 2^n\right)-\sum_{n=1}^k\epsilon_n2^n=2^{2003}+\sum_{n=k+1}^{2002}\varepsilon_n2^n &\geq&2^{2003}-\sum_{n=1}^{2002}2^n=2\gt 0, \end{eqnarray*} como queríamos demostrar.

Nota. Esta solución no es elemental, aunque es una aproximación válida y sencilla una vez se conoce la técnica de programación lineal. El mismo método sirve para probar el resultado cuando se cambia 2003 por cualquier otro entero positivo.

Si crees que el enunciado contiene un error o imprecisión o bien crees que la información sobre la procedencia del problema es incorrecta, puedes notificarlo usando los siguientes botones:
Informar de error en enunciado Informar de procedencia del problema

Sesión 2 —  Mar del Plata (Argentina), 17 de septiembre de 2003

Problema 770
Sea $M=\{1,2,\ldots,49\}$ el conjunto de los primeros 49 enteros positivos. Determinar el máximo entero $k$ tal que el conjunto $M$ tiene un subconjunto de $k$ elementos en el que no hay $6$ números consecutivos. Para este valor máximo de $k$, hallar la cantidad de subconjuntos de $M$ de $k$ elementos que tienen la propiedad mencionada.
pistasolución 1solución 2info
Pista. Demuestra que una forma de obtener el máximo es omitir los múltiplos de 6, pero no es la única ya que hay cierta flexibilidad en qué números se omiten.
Solución. Si quitamos los ocho múltiplos de $6$ nos quedan $41$ números que cumplen la propiedad ya que hemos dejado ocho bloques de 5 números consecutivos más el número 49 aislado. Veamos que si se omiten sólo 7 números, entonces tienen que quedar seis consecutivos, lo que nos dirá que $41$ es la respuesta. Si vemos estos siete números como separadore, los otros 42 números restantes quedarán separados en 8 grupos de números consecutivos. Si los 8 grupos tuvieran 5 o menos números, entonces tendríamos máximo $5\cdot 8=40$, pero hay 42, lo que nos dice que habrá 6 o más consecutivos (principio del palomar).

Veamos finalmente cuántos subconjuntos de $41$ elementos tienen la propiedad. Al igual que antes, al omitir 8 números, podemos transformar el problema en elegir 9 números $a_1,a_2,\ldots,a_9$ entre $1$ y $5$ que dirán la cantidad de números consecutivos que habrá entre cada uno de los 8 que se omiten. Como tiene que ser $a_1+a_2+\ldots+a_9=41$, podemos pensar en hacer $a_1=a_2=\ldots=a_9=5$ (el máximo) y luego restarle $1$ a cuatro de los nueve números (posiblemente con repetición). Esto no son más que combinaciones con repetición, que equivalen a su vez a elegir como separadores 9 elementos de un conjunto de $4+9=13$ elementos. Por tanto, tenemos un total de $\binom{13}{9}=\binom{13}{4}=715$ subconjuntos de $41$ elementos con la propiedad del enunciado.

Solución. Veamos en primer lugar cómo conseguir el máximo usando un razonamiento voraz. Supongamos que $k$ es el máximo y que $S$ es un subconjunto de $k$ elementos en el que no hay 6 números consecutivos. Tenemos dos propiedades importantes:
  • Si $S$ omitiera dos o más números consecutivos $n,n+1,\ldots,n+a$, entonces podríamos restarle $a$ unidades a todos los elementos de $S$ mayores que $n+a$ y tendríamos un conjunto $S'$ con el mismo número de elementos y también cumpliendo la propiedad. Si $n+a=49$, podríamos añadirle más números a partir del $n+1$ y tendríamos $S'$ con más elementos, luego $S$ no sería el conjunto de más elementos.
  • Si $S$ no contiene dos números $n$ y $n+a$ pero sí los intermedios $n+1,\ldots,n+a-1$, entonces tiene que ser $a\leq 6$ para que haya máximo cinco consecutivos. Si fuera $n+a\leq 6$, podemos añadir los números desde el $1$ hasta el $n$ y obtenemos un conjunto que cumple la propiedad con más elementos. Si fuera $a\lt 6$ (hay menos de 5 consecutivos), entonces podemos añadir $n+a$ al conjunto y sumarle una unidad a todos los elementos de $S$ mayores o iguales que $n+a$. Repitiendo el proceso, llegamos a un conjunto $S'$ con al menos el mismo número de elementos de $S$ y que tiene a $n+1,n+2,n+3,n+4,n+5$ pero no a $n$ ni $n+6$.

Estas idas nos dicen que un ejemplo de subconjunto $S\subset M$ con la mayor cantidad de números (sin 6 consecutivos) consiste en tomar 5 consecutivos, omitir el sexto, luego 5 consecutivos y omitir el sexto, y así sucesivamente. Como $49=8\cdot 6 +1$, tenemos que hay que omitir un mínimo de 8 números y que el mayor valor de $k$ es $49-8=41$.

Vamos ahora a contar cuántos subconjuntos de $41$ elementos tienen esta propiedad. La idea es que, al omitir 8 números, podemos transformar el problema en elegir 9 números $a_1,a_2,\ldots,a_9$ entre $1$ y $5$ que dirán la cantidad de números consecutivos que habrá entre cada uno de los 8 que se omiten. Como tiene que ser $a_1+a_2+\ldots+a_9=41$, podemos pensar en hacer $a_1=a_2=\ldots=a_9=5$ (el máximo) y luego restarle $1$ a cuatro de los nueve números (posiblemente con repetición). Esto no son más que combinaciones con repetición, que equivalen a su vez a elegir como separadores 9 elementos de un conjunto de $4+9=13$ elementos. Por tanto, tenemos un total de $\binom{13}{9}=\binom{13}{4}=715$ subconjuntos de $41$ elementos con la propiedad del enunciado.

Si crees que el enunciado contiene un error o imprecisión o bien crees que la información sobre la procedencia del problema es incorrecta, puedes notificarlo usando los siguientes botones:
Informar de error en enunciado Informar de procedencia del problema
Problema 768
En el cuadrado $ABCD$, sean $P$ y $Q$ puntos pertenecientes a los lados $BC$ y $CD$, respectivamente, distintos de los extremos y tales que $BP=CQ$. Se consideran puntos $X$ e $Y$, $X\neq Y$, pertenecientes a los segmentos $AP$ y $AQ$, respectivamente. Demuestre que, cualesquiera que sean $X$ e $Y$, existe un triángulo cuyos lados tienen las longitudes de $BX$, $XY$ y $DY$.
pistasolución 1info
Pista. Dobla el papel en el que has dibujado el cuadrado.
Solución. Si recortamos el cuadrado y también el triángulo $CPQ$, podemos doblar por las líneas $AP$ y $AQ$ para formar una pirámide con vértice en $A$. Observemos que, de esta forma $B$ se identifica con $D$ ya que ambos puntos están a la misma distancia de $A$. No obstante, para comprobar que efectivamente la pirámide se cierra, observamos que el triángulo de vértices $B=D$, $P$ y $Q$ en el espacio es rectángulo ya que es congruente con el triángulo $CPQ$ que hemos quitado. Ahora la solución al problema es evidente ya que $BX$, $XY$ y $DY$ forman un triángulo en el espacio.imagen
Si crees que el enunciado contiene un error o imprecisión o bien crees que la información sobre la procedencia del problema es incorrecta, puedes notificarlo usando los siguientes botones:
Informar de error en enunciado Informar de procedencia del problema
Problema 769
Se definen las sucesiones $\{a_n\}$ y $\{b_n\}$ como \begin{align*} a_0&=1,& a_{n+1}&=a_n^{2001}+b_n,\\ b_0&=4,& b_{n+1}&=b_n^{2001}+a_n, \end{align*} para todo $n\geq 0$. Demostrar que 2003 no divide a ninguno de los términos de estas sucesiones.
Sin pistas
Sin soluciones
info
Si crees que el enunciado contiene un error o imprecisión o bien crees que la información sobre la procedencia del problema es incorrecta, puedes notificarlo usando los siguientes botones:
Informar de error en enunciado Informar de procedencia del problema
José Miguel Manzano © 2010-2024. Esta página ha sido creada mediante software libre